第五章

第十三题

设$\left\{f_n\right\}$是(非空)完备度量空间$X$上的连续复函数序列,使$f(x)={\lim}f_n(x)$(作为一个复数)对每个$x{\in}X$都存在。

  1. 证明存在一个开集$V{\neq}\varnothing$和一个数$M<\infty$,对所有的$x{\in}V$及$n=1,2,3,\cdots,|f_n(x)|<M$。
  2. 若$\varepsilon>0$,证明存在一个开集$V{\neq}\varnothing$和一个整数$N$,当$x{\in}V$及$n{\ge}N$时,$|f(x)-f_n(x)|{\le}\varepsilon$。

提示:对$N=1,2,3,\cdots$,令

$$ A_N=\left\{x:|f_m(x)-f_n(x)|{\le}\varepsilon,当m{\ge}N和n{\ge}N时\right\} $$

因为$X={\bigcup}A_N$,于是某个$A_N$有非空内部。

证明:

1,设

$$ X_N=\left\{x{\in}X:|f_n(x)|{\le}N,当n=1,2,{\cdots}\right\} $$

则有$X={\bigcup}^{\infty}_{N=1}X_N$。

事实上,右边的并确实包含在左边,显然可得。

接下来我们证明$X{\subset}{\bigcup}X_N$,事实上,由于$f(x)={\lim}f_n(x)$,于是对于${\forall}x{\in}X$,存在$N>f(x)+1$使得当$n>N$时候有$f_n(x)<N$。因此$x{\in}X_N$,故此包含关系证明完毕。

从而我们知道有$X={\bigcup}X_N$。显然有$X_N$是闭集。

这是因为,设

$$ E_n=\left\{x{\in}X,|f_n(x)|{\le}N\right\} $$

则由于$[-N,N]$为闭区间,而$f_n$为连续函数,故此有$E_n$为闭集,而$X_N={\bigcap}^{\infty}_{n=1}E_n$。

由贝尔纲定理:已经知道$X$是非空完备空间,而$\left\{X_N\right\}$为$X$的闭子集序列,使得$X={\bigcup}X_N$,则存在$N_0$,使得$X_{N_0}{\neq}\varnothing$,也即有$Int(X_{N_0}){\neq}\varnothing$。令$V=Int(X_{N_0})$且$M=N_0+1$。,则可以推出:对所有的$x{\in}V$有$f_n(x)<M$。

2,设

$$ A_N=\left\{x:|f_m(x)-f_n(x)|{\le}\varepsilon,当m{\ge}N和n{\ge}N时\right\} $$

则有$X={\bigcup}A_N$。

事实上,右边的并确实包含在左边,显然可得。

接下来我们证明$X{\subset}X_N$,事实上,由于$f(x)={\lim}f_n(x)$,而这个其实等价于柯西列收敛,也就是$A_N$的定义。实际上,由于$f(x)={\lim}f_n(x)$,则对${\forall}\varepsilon,{\exists}N,x{\in}X$,当$m,n>N$时,有

$$ |f_m(x)-f(x)|<\frac{\varepsilon}{2},|f_n(x)-f(x)|<\frac{\varepsilon}{2}\\则|f_m(x)-f_n(x)|=|(f_m(x)-f(x))-(f_n(x)-f(x))|\\{\le}|f_m(x)-f(x)|+|f_n(x)-f(x)|<\frac{\varepsilon}{2}+\frac{\varepsilon}{2}=\varepsilon $$

于是则有$x{\in}A_N$。

从而我们知道有$X={\bigcup}A_N$。显然有$A_N$是闭集。

由贝尔纲定理:已经知道$X$是非空完备空间,而$\left\{A_N\right\}$为$X$的闭子集序列,使得$X={\bigcup}A_N$,则存在$N_0$,使得$A_{N_0}{\neq}\varnothing$,也即有$Int(A_{N_0}){\neq}\varnothing$。令$V=Int(A_{N_0})$。由于题设有$f(x)={\lim}f_n(x)$,故取$m{\to}\infty$于是则有

$$ |f(x)-f_n(x)|{\le}\varepsilon $$

于是则有对于所有的$x{\in}V,n>N$,有$|f(x)-f_n(x)|{\le}{\varepsilon}$成立。

第十八题

设$\left\{\Lambda_n\right\}$是一个赋范线性空间$X$到巴拿赫空间$Y$的有界线性变换的序列。对所有$n$,$\|{\Lambda}\|{\le}M<\infty$,并设有一个稠密集$E{\subset}X$,对每个$x{\in}E,\left\{\Lambda_nx\right\}$都收敛。证明对每一个$x{\in}E,\left\{\Lambda_nx\right\}$都收敛。

证明:因为$E{\subset}X$是稠密集,于是对于任意的$x{\in}X$,存在一个序列$\left\{x_k\right\}{\subset}E$使得$\|x_k-x\|{\to}0(k{\to}\infty)$。并且,对于任意的$k{\in}\mathbb{N}$,有$\left\{{\Lambda}_nx_k\right\}{\subset}Y$收敛。因此存在序列$\left\{y_k\right\}{\subset}Y$使得

$$ {\Lambda}_nx_k{\to}y_k,n{\to}\infty,k=1,2,{\cdots} $$

对于任意的$\varepsilon>0$,存在$N>0$使得

$$ \|x_k-x_{k'}\|<\frac{\varepsilon}{M}{\quad}{\forall}k,k'>N $$

且有

$$ \|y_k-y_{k'}\|=\lim_{n{\to}\infty}\|{\Lambda}_n(x_k-x_{k'})\|{\le}\lim_{n{\to}\infty}\|{\Lambda}_n\|{\cdot}\|x_k-x_{k'}\|{\le}M{\cdot}\frac{\varepsilon}{M}=\varepsilon $$

因此意味着$\left\{y_k\right\}{\subset}Y$是柯西序列。因为$Y$是巴拿赫空间(完备性),存在$y{\in}Y$并且$\|y_k-y\|{\to}0$。因此

$$ \begin{align*} \|{\Lambda_nx-y}\|&=\|{\Lambda}_nx-{\Lambda}_nx_k+{\Lambda}_nx_k-y_k+y_k-y\|\\ &{\le}\|{\Lambda}_n\|{\cdot}\|x_k-x\|+\|{\Lambda}_nx_k-y_k\|+\|y_k-y\|\\ &{\le}M{\cdot}\|x_k-x\|+\|{\Lambda}_nx_k-y_k\|+\|y_k-y\| \end{align*} $$

也就是有$\lim_{n{\to}\infty}{\Lambda}_nx=y$。

第二十二题

设$f{\in}C(T)$且对某个$\alpha>0,f{\in}Lip{\alpha}$,证明$f$的傅立叶级数收敛到$f(x)$。考虑$f(0)=0$的情形就足够了。部分和$s_n(f;0)$与积分$\frac{1}{\pi}\int^{\pi}_{-\pi}f(t)\frac{sinnt}{t}dt$的差在$n{\to}\infty$时趋于0,函数$f(t)/t$属于$L^1(T)$。应用黎曼-勒贝格引理。实际上可以证明这个级数在$T$上一致收敛。

证明:

首先:如果$f{\in}C(T)$且$f{\in}Lip\;{\alpha}$,则

$$ |\frac{f(t)}{t}|=\frac{|f(t)|}{|t|^{\alpha}|t|^{1-{\alpha}}}{\le}M_f|t|^{\alpha-1} $$

其中$M_f$和第十一题定义的一样。因为$M_f|t|^{\alpha-1}$在$L^1(T)$中,$\frac{f(t)}{t}$在$L^1(T)$中。应用黎曼-勒贝格定理,我们能得到

$$ \frac{1}{\pi}\int^{\pi}_{-\pi}f(t)\frac{sinnt}{t}dt{\to}0,\frac{1}{\pi}\int^{\pi}_{-\pi}f(t)\frac{cosnt}{t}dt{\to}0 {\quad}当n{\to}\infty $$

由课本5.11(9),我们知道

$$ s_n(f;0)=\frac{1}{2\pi}\int^{\pi}_{-\pi}f(t)\frac{sin(n+\frac{1}{2})t}{sin\frac{t}{2}}dt\\=\frac{1}{\pi}\int^{\pi}_{-\pi}\frac{f(t)}{t}\frac{\frac{t}{2}}{sin\frac{t}{2}}sin(n+\frac{1}{2})tdt $$

注意到

$$ sin(n+\frac{1}{2})t=sinntcos(\frac{t}{2})+cosntsin(\frac{t}{2})\\ f(t)sinntcos(\frac{t}{2})sin(\frac{t}{2})=(\frac{f(t)}{t})(\frac{t}{sin(\frac{t}{2})})cos(\frac{t}{2})\\ 且(\frac{f(t)}{t})(\frac{t}{sin(\frac{t}{2})})cos(\frac{t}{2}){\in}L^1(\mu) $$

于是有

$$ s_n(f;0)=\frac{1}{\pi}\int^{\pi}_{-\pi}\frac{f(t)}{t}\frac{\frac{t}{2}}{sin\frac{t}{2}}cos(\frac{t}{2})sinntdt+\frac{1}{2\pi}\int^{\pi}_{-\pi}f(t)cosntdt $$

由于$f{\in}L^1(\mu)$,因此可以再次用黎曼-勒贝格引理,则因为

$$ \frac{1}{2\pi}\int^{\pi}_{-\pi}\frac{sin(n+\frac{1}{2})t}{sin\frac{t}{2}}dt =\frac{1}{2\pi}\int^{\pi}_{-\pi}D_n(t)dt =\frac{1}{2\pi}\int^{\pi}_{-\pi}\sum^n_{k=-n}e^{ikt}dt=1 $$

当$n{\to}\infty$时,有$s_n(f;0){\to}0$。

对任意的$x_0{\in}T$,有

$$ s_n(f;x_0)-f(x_0)=\frac{1}{2\pi}\int^{\pi}_{-\pi}(f(t-x_0)-f(x_0))\frac{sin(n+\frac{1}{2})t}{sin\frac{t}{2}}dt $$

令$g(x)=f(x-x_0)-f(x_0)$。于是有$g(0)=0,g{\in}C(T)$且$g{\in}Lip{\;}\alpha$。由上述的证明我们得到:当$n{\to}\infty$时,有$s_n(g;0){\to}0$。因此$s_n(f;x_0)-f(x_0)=s_n(g;0){\to}0$。

接下来我们证明$|s_n(f;x)-f(x)|{\rightrightarrows}0$在$T$上成立。

令$x,y{\in}T$,

$$ I_1=\left\{t{\in}T:t<|x-y|\right\}\\I_2=\left\{t{\in}T:t>|x-y|\right\} $$

$$ |s_n(f;x)-f(x)-(s_n(f;x)-f(y))|\\{\le}\frac{1}{2\pi}\int^{\pi}_{-\pi}|f(x-t)-f(x)-f(y-t)+f(y)|{\cdot}|D_n(t)|dt $$

由于$f{\in}Lip{\;}\alpha$,于是有

$$ |f(x-t)-f(y-t)+f(y)-f(x)|{\le}2M_f|x-y|^{\alpha}\\ 且|f(x-t)-f(x)-f(y-t)+f(y)|{\le}2M_f|t|^{\alpha} $$

因为

$$ \lim_{t{\to}0}\frac{t}{sint}=1,且|\frac{\frac{t}{2}}{sin(\frac{t}{2})}|在T上有界 $$

于是存在$M>0$,使得$|\frac{\frac{t}{2}}{sin(\frac{t}{2})}|<M$。

$$ \begin{align*} \int_{I_1}|f(x-t)-f(x)-f(y-t)+f(y)|{\cdot}|D_n(t)|dt&{\le}2M_f\int_{I_1}|t|^{\alpha}|D_n(t)|dt\\ &{\le}4M_f\int_{I_1}|t|^{\alpha-1}dt\\ &{\le}\frac{4M_f}{{\alpha}|x-y|^{\alpha}} \end{align*} $$

$$ \begin{align*} \int_{I_2}|f(x-t)-f(x)-f(y-t)+f(y)|{\cdot}|D_n(t)|dt&{\le}2M_f\int_{I_2}|x-y|^{\alpha}2M|\frac{1}{t}|dt\\ &{\le}4M_fM|x-y|^{\alpha}ln|x-y| \end{align*} $$

因为当$x{\to}y$时,有$|x-y|^{\alpha}ln|x-y|{\to}0$。于是对于${\forall}\varepsilon>0$,存在$\delta>0$,如果有$|x-y|<\delta$,则

$$ |s_n(f;x)-f(x)-(s_n(f;y)-f(y))|<\varepsilon $$

由上述的证明过程,$\delta$并不依赖于$n,x,y$。特别地$|s_n(f;x)-f(x)|$是在$T$上等度连续的。

对于所有的$x{\in}T,{\forall}\varepsilon>0$,存在一个$N_x>0$,使得如果有$n>N_x$,则$|s_n(f;x)-f(x)|<\varepsilon$。由于等度连续性质,存在一个开球$B(x,\delta)$使得对于$x{\in}B(x,\delta)$,有$|s_n(f;x)-f(x)|<\varepsilon$。

而集列$\left\{B(x,\delta)\right\}$是$T$的开覆盖,由于$T$是紧的,于是可以找到有限的子覆盖$B(x_1,\delta),\cdots,B(x_k,\delta)$覆盖了$T$,令$N=\max\left\{N_{x_1},\cdots,N_{x_k}\right\}$。则如果有$n>N,|s_n(f;x)-f(x)|<\varepsilon$对所有的$x{\in}T$都成立。

故此有$s_n(f;x)$在$T$上一致收敛到$f(x)$。

Last modification:December 26th, 2019 at 10:47 pm
如果觉得我的文章对你有用,请随意赞赏